What which of these numbers are divisible by 13 is it a. 52 or b. 9 c. 30 d. 27?

1 Answer

Answer :

a.

Related questions

Description : 1, 2, 3, 4, 5, 6, 7, 8, 9, 10, 11, 12, 13, 14, 15, 16, 17, 18, 19, 20, 21, 23, 24, 25, 26, 27, 28, 29, 30, 31, 32, 33, 34, 35, 36, 37, 38, 39, 40, 41, 42, 43, 44, 45, 46, 47, 48, 49, 50'. What number is missing? -Riddles

Last Answer : 22

Description : The population of Khyber Pakhtunkhwa according to 2017 Census is: A. 27 Million B. 33.44 Million C. 15.54 Million D. 30.52 Million

Last Answer : ANSWER: D

Description : KPK The population of Khyber Pakhtunkhwa according to 2017 Census is: A. 27 Million B. 33.44 Million C. 15.54 Million D. 30.52 Million

Last Answer : ANSWER: D

Description : The population of Khyber Pakhtunkhwa according to 2017 Census is: A. 27 Million B. 33.44 Million C. 15.54 Million D. 30.52 Million

Last Answer : ANSWER: D

Description : How many numbers between 16 to 189 are divisible by 9 but not by 4?

Last Answer : There are 14 such numbers between 16 and 189.

Description : What numbers are divisible by 3 9 6?

Last Answer : Any number of the form 18*k where k is an integer.

Description : Consider the example of finding the probability of selecting a red card or a 9 from a deck of 52 cards. A) 15/26 B) 26/15 C) 7/13 D) 13/7

Last Answer : Answer: C) We need to find out P(R or 6) Probability of selecting a Red card = 26/52 Probability of selecting a 9 = 4/52 Probability of selecting both a red card and a 9 = 2/52  P(R or 9) = P(R) + P(9) – P(R and 9) = 26/52 + 4/52 – 2/52 = 28/52 = 7/13.

Description : When `._(13)^(27)Al` is bombarded whith `alpha`-particles, a radioactive isotope of phosphorus `._(15)^(30)P` with the emission of …. Is formed

Last Answer : When `._(13)^(27)Al` is bombarded whith `alpha`-particles, a radioactive isotope of phosphorus ` ... A. neutrons B. protons C. positrons D. electrons

Description : The ages of Gaurav and Goutham will be in the ratio 5 : 7 after ten years from now and will be in the ratio 13 : 18 after twelve years from now. Find the ratio of the sum of their ages 10 years hence to the sum of their ages 12 ... hence. 1 : 25 : 27 2 : 13 : 15 3 : 14 : 17 4 : 18 : 23 5 : 30 : 31

Last Answer : 5 : 30 : 31

Description : Maximum slenderness ratio for load-bearing masonry wall in cement mortar, as per IS code, shall not exceed. (a) 13 (b) 20 (c) 27 (d) 30

Last Answer : (c) 27

Description : In 165 litres of mixtures of milk and water, water is only 28%. The milkman sold 40 litres of this mixture and then he added 30 litres of pure milk and 13 litres of pure water in the remaining mixture. What is the percentage ... mixture? 1) 29.35% 2) 28. 57% 3) 24. 57% 4) 27. 75% 5) 26. 57%

Last Answer : 2) 28. 57%

Description : A square surface 3 m × 3 m lies in a vertical line in water pipe its upper edge at water surface. The hydrostatic force on square surface is (A) 9,000 kg (B) 13,500 kg (C) 18,000 kg (D) 27,000 kg

Last Answer : Answer: Option B

Description : The cash difference between the selling price of an article at a profit of 8% and 4% is Rs 3 the ratio of the two selling price is A. 51 : 52 B. 27 : 26 C. 51 : 53 D. 52 : 55

Last Answer : Answer – B (27:26) Explanation – Let the C.P of the article be Rs. x Then required ratio = 108% of x / 104% of x = 108/104= 27:26

Description : Find out the wrong number in the sequence. 52, 51, 48, 43, 34, 27, 16 (A) 27 (B) 34 (C) 43 (D) 48

Last Answer : Answer: B The series is decreese with order 1, 3, 5, 7, 9, 11. Therefore expected series will be 52, 51, 48, 43, 36, 27, 16. Hence 34 is wrong in seriese there must be 36.

Description : What least number would be subtracted from 427398 so that the remaining number is divisible by 15? 1. 13 2. 3 3. 16 4. 11 5. 14

Last Answer : Answer- 2 ( 3) Explanation:- On dividing 427398 by 15 we get the remainder 3, so 3 should be subtracted 

Description : The average of four consecutive even numbers is 27. The largest of these numbers is: (A) 36 (B) 32 (C) 30 (D) 28 

Last Answer : Answer: C Four consecutive even number = x, x+2,x+4,x+6. According to question, x+x+2x+4x+6/4=27. 4x+12/4 = 27 4x =108 - 12=96. x=96/4=24. Hence , The largest number =x+4= 24+6=30. The numbers are 24, 26 28, and 30 , which addition is 108 which average is 27, So biggest is 30

Description : What numbers has three decimal places for 27.9?

Last Answer : It is simply 27.900

Description : What numbers has three decimal places for 27.9?

Last Answer : It is simply 27.900

Description : 20

Last Answer : 20 numbers from 1-100 are divisible by 5.

Description : What Number are divisible by 8 can be also divisible by what numbers?

Last Answer : They can also be evenly divided by 1, 2 and 4.

Description : What numbers are divisible by 4 or 6?

Last Answer : 12 and any multiples of 12

Description : Which of these numbers are divisible by 3 - a. 29 b. 16 c. 24 d. 14?

Last Answer : Only c. 24 is evenly divisible by 324/3 =- 8

Description : What is 519 divisible by in whole numbers?

Last Answer : It is divisible by any of its factors which are 1, 3, 173 and519

Description : What which of these numbers is divisible by 12 is it a. 6 b. 26 c. 12 d. 42?

Last Answer : c.

Description : What numbers are 21 divisible by?

Last Answer : They are members of the infinite set of numbers of the form 21*kwhere k is an integer.

Description : The least number that is divisible by all the numbers from 1 to 5 is: (a) 70 (b) 60 (c) 80 (d) 90

Last Answer : (b) 60

Description : The odd numbers from 1 to 45 which are exactly divisible by 3 are arranged in an ascending order. The number at 6 th position is (A) 18 (B) 24 (C) 33 (D) 36

Last Answer : Answer: C

Description : What number does 13 and 52 go into?

Last Answer : Any multiple of 52.

Description : How do i do this question The gradient of the line joining (-13) to (pq) is -2. The gradient of the line joining (pq) to (52) is -1. Calculate the values of p and q?

Last Answer : If you mean point of (-1, 3) with a gradient of -2 and point (5,2) with a gradient of -1 then as straight line equations they workout as y = -2x+1 and y = -x+4 respectively.As to the values of p and q not enough information has beengiven.

Description : What is a healthy weight range for a 5'2" 13 year old girl?

Last Answer : While most girls are different there are some guidelines that most can follow. At this age the girl should be between 95-110 depending on the body frame. She should not be focusing on dieting at this age.

Description : How many times can 13 go into 52?

Last Answer : Hhjj

Description : Find the probability of getting a spade. A. 1/26 B. 1/13 C. 1/52 D. 1/4

Last Answer : D. 1/4

Description : Find the probability of getting a red face card. A. 3/26 B. 1/13 C. 1/52 D. 1/4

Last Answer : A. 3/26

Description : The distance of the conductor when the area and length of the conductor is 24m2 and 13.56m. a) 1.76 b) 2.67 c) 1.52 d) 2.15

Last Answer : a) 1.76

Description : If the potential is given by, V = 10sin θ cosφ/r, find the density at the point P(2, π/2, 0) (in 10 -12 units) a) 13.25 b) 22.13 c) 26.31 d) 31.52

Last Answer : b) 22.13

Description : There are 4 borthers 3 of them want to buy a television which is 30 pounds. Each of them pay 10 pound each they give the money to the fourth brother and asked him to go into town and buy the television. ... So the 3 brothers only payed 9 pounds each. 3 times 9 is 27 were is the last pound? -Riddles

Last Answer : Add 3 on to 25 = 28 then add 2

Description : Standard free energy (∆G°) of hydrolysis of ADP to AMP + Pi is (A) –43.3 KJ/mol (B) –30.5 KJ/mol (C) –27.6 KJ/mol (D) –15.9 KJ/mol

Last Answer : Answer : C

Description : Two trains are running at 40 km/hr and 20 km/hr respectively in the same direction. Fast train completely passes a man sitting in the slower train in 5 seconds. What is the length of the fast train? A.27 7/9 m B.28 m C.29 m D.30 2/7 m E.None of these 

Last Answer : Answer – A (27 7/9 m) Explanation – When SAME direction- MINUS Relative speed = (40-20) km/hr = [20 x 5/18] m/sec = [50/9] m/sec. Length of faster train = sxt=[50/9 x 5] m = 250/9 m = 27 7/9 m.

Description : Age of father 30 years ago was 9 times the age of her son. After 30 years, father’s age will be thrice that of his son. Find the ratio of their present ages. a) 21 : 5 b) 29 : 5 c) 27 : 4 d) 21 : 3

Last Answer : A At age of son be x and as father's age is 9 times the age of her son, let it be 9x, nine years ago. At present: father's age will be (9x + 30) and son's age will be (x + 30) After 30 years: father's age ... x = 20 We are asked to find the present ratio. (9x + 30) : (x + 30) = 210 : 50 = 21:5

Description : What even number is graeter than 20 less than 30 and divisible by 3?

Last Answer : 7

Description : The least number which should be added to 2497 so that the sum is exactly divisible by 5, 6, 4 and 3 is: A.10 B.14 C.23 D.30 E.None of these

Last Answer : Answer – C (23) Explanation – L.C.M. of 5, 6, 4 and 3 = 60. On dividing 2497 by 60, the remainder is 37. Number to be added = (60 – 37) = 23

Description : What number is divisible by 9 123 234 or 345?

Last Answer : 123

Description : Is 646 divisible by 9?

Last Answer : 6 + 4 + 6 = 161 + 6 = 7→ No; 646 is not divisible by 9 (there is a remainder of 7).-----------------------------------------Only if the sum of the ... ifthis single digit is 9 is the original number divisible by 9,otherwise this single digit is the remainder when the originalnumber is divided by 9.

Description : Is 4932 divisible by 9?

Last Answer : Yes

Description : Is 90 divisible by 2 3 4 5 6 9 10?

Last Answer : Yes, all except 4

Description : What are all of the different ways to choose the tens digit A and the ones digit B in the number 631872AB so that the number will be divisible by 9?

Last Answer : A + b = 0, 9, 18(9,0)(8,1)(7,2)(6,3)(5,4)(4,5)(3,6)(2,7)(1,8)(0,9)(0,0)(9,9)-------------------------------------------To be divisible by 9, the sum 6 + 3 + 1 + 8 + 7 + 2 + A + B = 27+ A ... , 5),(5, 4), (6, 3), (7, 2), (8, 1), (9, 0)Note that (0, 0) must not be forgotten as 27 + 0 + 0 = 27 = 3 9.

Description : What are all of the different ways to choose the ones digit A in 271854A so that the number will be divisible by 9?

Last Answer : A can be 0 or 9.

Description : Why the divisibility test for 9 is valid and a way to determine whether a three-digit counting number ABC is divisible by 9?

Last Answer : If the sum of digits add up to 9 then the number is divisible by 9 as for example the digits of 450 add up to 9 and so 450/9 = 50-------------------------------------- ... sum is 9, then the original number is divisible by 9, otherwise it is the remainder when the original number is divided by 9.

Description : Is 90 divisible by 5 and 9?

Last Answer : Yes. 90 is divisible by 9, 10 times. 90 is divisible by 5, 18times.

Description : Why the divisibility test for 9 is valid and a way to determine whether a three-digit counting number ABC is divisible by 9?

Last Answer : If the sum of digits add up to 9 then the number is divisible by 9 as for example the digits of 450 add up to 9 and so 450/9 = 50-------------------------------------- ... sum is 9, then the original number is divisible by 9, otherwise it is the remainder when the original number is divided by 9.